LSAT and Law School Admissions Forum

Get expert LSAT preparation and law school admissions advice from PowerScore Test Preparation.

 Administrator
PowerScore Staff
  • PowerScore Staff
  • Posts: 8916
  • Joined: Feb 02, 2011
|
#81136
Complete Question Explanation

Parallel Reasoning. The correct answer choice is (B).

Answer choice (A):

Answer choice (B): This is the correct answer choice.

Answer choice (C):

Answer choice (D):

Answer choice (E):

This explanation is still in progress. Please post any questions below!
 deck1134
  • Posts: 160
  • Joined: Jun 11, 2018
|
#48963
Hi PowerScore Staff,

I used the Test of Abstraction to solve this problem. Is that the right method? I noticed that the structure is "It has three, because it does not have two, it must have the remaining one," which led to B. But I was uncomfortable with this abstraction. Is there a better way to think about this problem?
User avatar
 Dave Killoran
PowerScore Staff
  • PowerScore Staff
  • Posts: 5852
  • Joined: Mar 25, 2011
|
#49169
Hi Deck,

Thanks for the question! This problem is conditional in nature, but I find it much easier to do it using Abstraction. And, your abstraction perfectly describes what is occurring, so it an excellent attack method.

Overall, this is a good example of where you can see exactly what they are doing in a broad sense, and that allows you to crush this problem quickly using the right tool. your Abstraction was spot on—don't doubt yourself!
User avatar
 mrdmass725
  • Posts: 9
  • Joined: Feb 21, 2021
|
#84369
Correct me if I am wrong but from my understanding C is wrong because it equates Vaccine X to every Vaccine which would be like saying the AR3000 would be equated to every computer as opposed to the right answer C which equates Vac X to only commonly use vaccines which is a subset of all possibly used vaccines in the same way that computers with intelligence is a subset of all computers. Am I wrong in my reasoning here?
 Jeremy Press
PowerScore Staff
  • PowerScore Staff
  • Posts: 1000
  • Joined: Jun 12, 2017
|
#84391
Hey mrdmass,

Yes, that's exactly right!

In answer choice C, we already know that Vaccine X is a vaccine, so the conclusion can be more definitive (it it doesn't have the two characteristics it must have the third). This contrasts with the stimulus, where we do not know that the AR3000 is an "intelligent" computer, and answer choice B, where we do not know if Vaccine X is a "commonly used" vaccine. Both the stimulus and answer choice B then have more tentative conclusions: if it's going to fit the category (intelligent or commonly used), which we don't know yet, it must have the third characteristic.

There's another contrast where, in answer choice C, we don't actually know from the premises which characteristics Vaccine X does not have. The conclusion just asserts (validly) that if it doesn't have two characteristics, it has the third. This contrasts with the stimulus and answer choice B, where we know for sure that the AR3000 does not have two of the three characteristics, and where we know for sure that Vaccine X does not have two of the three characteristics.

Nice work!
User avatar
 landphil
  • Posts: 42
  • Joined: Jul 01, 2022
|
#98650
The stimulus never clarifies that AR3000 is a computer, it just adds that "if it is intelligent" at the end. I put E because E makes the same error I thought. The error being it equates "Vaccine X" with "every commonly used vaccine." Just like how the stimulus assumes an intelligent AR3000 is an intelligent computer.

Can you please explain why B is a better answer than E in light of this?
User avatar
 Jeff Wren
PowerScore Staff
  • PowerScore Staff
  • Posts: 385
  • Joined: Oct 19, 2022
|
#98722
Hi landphil,

You're right that the stimulus never states that the "AR3000" is a computer; it is only implied based on the name/model# itself and the context of the argument. While this certainly could be a problem and you were right to pick up on it, in this case it turns out to be a bit of a red-herring.

For parallel reasoning questions, as the name implies, you're looking for the answer that most closely matches the underlying reasoning even if it is not an exact match.

In the stimulus, we are told that for a computer to be intelligent, it must possess at least one of three qualities. The AR3000 (which is assumed here to be a specific computer model), doesn't have two of the three qualities. The conclusion is that in order for it to be intelligent it would need to have the third quality.

Answer E starts off similar with three possible types of every commonly used virus. It rules out one of the three types for vaccine X, and then concludes that if it isn't a second type, it must be the third type.

The key difference between answer E and the stimulus is that the stimulus rules out 2 of the 3 options in the premises, while answer E only rules out one. Also, the conclusion in the stimulus does refer back to the original sufficient condition "if it is intelligent" even though it would have been more clear to specify "intelligent computer."
User avatar
 landphil
  • Posts: 42
  • Joined: Jul 01, 2022
|
#102246
Jeff Wren wrote: Wed Jan 04, 2023 2:02 pm Hi landphil,

You're right that the stimulus never states that the "AR3000" is a computer; it is only implied based on the name/model# itself and the context of the argument. While this certainly could be a problem and you were right to pick up on it, in this case it turns out to be a bit of a red-herring.

For parallel reasoning questions, as the name implies, you're looking for the answer that most closely matches the underlying reasoning even if it is not an exact match.

In the stimulus, we are told that for a computer to be intelligent, it must possess at least one of three qualities. The AR3000 (which is assumed here to be a specific computer model), doesn't have two of the three qualities. The conclusion is that in order for it to be intelligent it would need to have the third quality.

Answer E starts off similar with three possible types of every commonly used virus. It rules out one of the three types for vaccine X, and then concludes that if it isn't a second type, it must be the third type.

The key difference between answer E and the stimulus is that the stimulus rules out 2 of the 3 options in the premises, while answer E only rules out one. Also, the conclusion in the stimulus does refer back to the original sufficient condition "if it is intelligent" even though it would have been more clear to specify "intelligent computer."
Yes this is all true thank you for the coherent and helpful explanation. My LSAT brain at the time had not quite learned to process which differences between stimulus and answer choice were important and which were not. Thank you for your patience.

Get the most out of your LSAT Prep Plus subscription.

Analyze and track your performance with our Testing and Analytics Package.